You are on page 1of 93

LEGAL MEDICINE & MEDICAL JURISPRUDENCE

C 1. One of the following is not helpful in determining the sex of


the skeleton:
A. Skull
B. Sternum
C. Cranial sutures
D. Pelvis
Chapter IIIMedico Legal Aspects of Identification; p.80
Legal Medicine by Pedro Solis (1987 ed)

B 2. The medico legal classification of death due to injuries


inflicted upon the body by some form of outside force that is
the proximate cause thereof is:
A. Natural
B. Violent
C. Sickness caused
D. Instantaneous
Chapter VICause of Death; p.183
Legal Medicine by Pedro Solis (1987 ed)

C 3. The body should be buried within 12 hours if this condition


is present:
A. Body was not embalmed
B. Not subject of legal investigation
C. Death due to dangerous communicable disease
D. Death due to toxic fumes
Chapter VIIIDisposal of thee Dead Body; p.221
Legal Medicine by Pedro Solis (1987 ed)
A 4. Pedro attacked Juan using a bladed weapon. Instinctively
Juan tried to fend of the attack and suffered wounds as a result
of self-protection. What special type of wound did he suffer?
A. Defense
B. Self-inflicted
C. Offense

1
D. Patterned

Chapter IXMedico Legal Aspects of Physical Injuries;


p.244-245
Legal Medicine by Pedro Solis (1987 ed)

C 5. Josefa does not intend to hurt her child, Jose. When she is
drunk, she periodically batters Jose. She becomes
remorseful afterwards and provides proper care in between
her drinking spree. What type of child abuser is Josefa?
A. Occasional child abuser
B. Constant child abuser
C. Intermittent child abuser
D. One time child abuser

Chapter XX-Child Abuse or Neglected Child; p.480


Legal Medicine by Pedro Solis (1987 ed)

A 6. Josefa is 10 years old. She gave consent to have carnal


knowledge with a male tourist provided she is paid money.
Which is true?
A. Josefa is a victim of rape.
B. Josefa is a child prostitute and thus no rape
occurred.
C. There is no rape since Josefa consented.
D. Josefa must be 12 years old to be a victim of
statutory rape

Chapter XXI-Medico-Legal Aspects of Sex Crimes; p.500


Legal Medicine by Pedro Solis (1987 ed)

C 7. Pedro calls people up on the phone in the early hours of


the morning and starts using obscene and foul language to
obtain sexual gratification. This sexual deviation is known
as:

2
A. Pedophila
B. Masochism
C. Coprolalia
D. Exhibitionism

Chapter XXI-Medico-Legal Aspects of Sex Crimes; p.536


Legal Medicine by Pedro Solis (1987 ed)

D 8. Artificial donor insemination with the consent of the husband but


without the consent of the wife may result in:
A. Charges of rape against the physician
B. Action for divorce against the husband based on assault
C. A paternity suit against the donor
D. Criminal assault against the physician

C 9. Pregnancy may be a ground for the annulment of marriage


when:
Husband at the time of marriage knows the child is not his
Wife conceals the pregnancy caused by her husband
Wife conceals the fact the child is that of a man not her husband
The husbands files for annulment 10 years from the discovery
the child is not his

Chapter XXII-Pregnancy; p.540


Legal Medicine by Pedro Solis (1987 ed)

A 10. Which of the following type of abortion, whether


performed by a health professional or not, is criminal and
punishable by law?
A. Intentional
B. Spontaneous
C. Consequence of a therapeutic act
D. Threatened

Chapter XXIV-Abortion; p.559

3
Legal Medicine by Pedro Solis (1987 ed)

A 11. To obtain legal personality, a 27-week AOG infant must live


for at least:
A. 24 hours
B. 72 hours
C. 48 hours
D. None of the above

Chapter XXV-Birth; p.575


Legal Medicine by Pedro Solis (1987 ed)

A 12. A female schizophrenic patient in a lucid state pushed a


woman she did not know was pregnant down the stairs causing
her to abort. The patient is liable for:
A. Unintentional abortion
B. Intentional abortion
C. Accidental abortion
D. Not liable

Chapter XXIV-Abortion; p.559


Legal Medicine by Pedro Solis (1987 ed)

C 13. For a child to be legitimate, he must be born in lawful


wedlock within these number of days after the dissolution
of the previous marriage:
A. 120 days
B. 180 days
C. 300 days
D. 280 days

Chapter XXVII-Paternity and Fiilation; p.588


Legal Medicine by Pedro Solis (1987 ed)

4
B 14. Chingoy and Maria are Filipino Chinese with no history on both
sides of thefamily of ancestors who are black. They got
married and a child with dark skin, wiry hair and thick lips was
born. The child is:
A. Presumed legitimate
B. Presumed illegitimate based on ethnic reason
C. Definitely legitimate
D. Definitely illegitimate

Chapter XXII-Paternity and Filiation; p.592


Legal Medicine by Pedro Solis (1987 ed)

C 15. A man who had a sex change operation declared thereafter he


was a woman. A marriage was contracted with a boyfriend of 10
years and solemnized by a minister who was aware of the
change operation. The marriage is:
A. Valid
B. Null and void upon declaration by a court of law
C. Void from the start
None of the above

Chapter XXIX-Medico-Legal Aspects of Marital Union and


Dissolution; p.613
Legal Medicine by Pedro Solis (1987 ed)

B 16. Dr. Pedro is single but keeps a married woman in his


condominium. Which is true?
A. Dr. Pedro is liable for concubinage.
B. Dr. Pedro is liable for adultery.
C. The mistress is liable for concubinage.
D. Both are not liable.

Chapter XXIX-Medico-Legal Aspects of Marital Union and


Dissolution; p.619
Legal Medicine by Pedro Solis (1987 ed)

5
A 17. An idiot is exempted from criminal liability. An idiot has an
IQ of:
A. 0 20
B. 40 70
C. 20 40
D. 70 80

Chapter XXXII-Disturbance of Mentality; p.643


Legal Medicine by Pedro Solis (1987 ed)

C. 18. Jose was prescribed a psychotropic agent for his


depression. He developed an idiosyncratic reaction with
hallucinations. He killed his spouse thinking that she was a
three-headed monster. One of the following is applicable in this
case:
A. Jose is guilty of homicide
B. The psychiatrist who prescribed the drug is guilty of
malpractice
C. Jose can claim temporary insanity as a defense
D. The physician and Jose are liable

Chapter XXXII-Disturbance of Mentality; p.627


Legal Medicine by Pedro Solis (1987 ed)

C 19. The most widely accepted method to measure alcohol and


determine drunkenness is:
A. Urine alcohol conc.
B. Breath alcohol conc.
C. Blood alcohol conc.
D. None of the above

Chapter XXXIII-Alcoholism; p.707


Legal Medicine by Pedro Solis (1987 ed)

6
C 20. The body vested by law to have general supervision and
regulation over the practice of medicine is:
A. Commission on Higher Education
B. Department of Education
C. Professional Regulatory Commission

D. Board of Medical Education

D 21. The specific body that exercise the power of regulation


and supervision over medical education is:
A. Commission on Higher Education
B. Department of Education
C. Association of Phil. Medical Colleges
D. Board of Medical Education

Chapter II - Licensure and Regulatory Laws; pp.8-9


Medical Jurisprudence by Pedro Solis (1988 ed)

C 22. Any person who performs any of the acts constituting


practice of medicine but is not qualified to do so shall be:
A. Committing malpractice
B. Committing fraud
C. Illegally practicing medicine
D. Considered not to be practicing medicine

Chapter III The Practice of Medicine; pp.38, 48


Medical Jurisprudence by Pedro Solis (1988 ed)

B 23. Pedro Doctor graduated from the Medical School and


immediately used the title MD after his name. He is:
A. Committing malpractice
B. Academically entitled to use MD
C. Illegally practicing medicine
D. Falsely using MD after his name

7
Chapter III The Practice of Medicine; p.48
Medical Jurisprudence by Pedro Solis (1988 ed)

B 24. A 60-year old male with prostatic cancer consented in writing to


undergo treatment under a new drug. The patient continued to
deteriorate and sued the physician. The physician was absolved
under the doctrine of:
A. Common knowledge
B. Assumption of Risk
C. Contributory negligence
D. Sole responsibility

Chapter X Doctrines Applied in Malpractice Cases;


pp.253-254
Medical Jurisprudence by Pedro Solis (1988 ed)

D 25. Patient -physician relationship is not terminated when:


A. Patient recovers from the condition being treated
B. Physician considers his further service will no longer be
beneficial to the patient
C. Physician discharges the patient
D. Physician abandons the patient

Chapter IV Physician-Patient Relationship; p.77


Medical Jurisprudence by Pedro Solis (1988 ed)

D 26. A physician issued a medical certificate in favor of a friend


who was absent from work attesting the latter was sick when in
fact he was not. The physician under the law is liable:
A. Civilly
B. Administratively
C. Morally
D. Criminally

Chapter XVI Liabilities of Physicians; p.187

8
Medical Jurisprudence by Pedro Solis (1988 ed)

27. Under article III of the Medical Act, citizens of other countries
maybe allowed to take the Philippine Medical board examination
if
A. they promise to practice in the Philippines
B. their countries allow Filipinos to practice there
C. they give substantial donation to the Philippine
government
D. there is lack of Filipino doctors in the country

28. A physician who is subpoenaed by a trial court but


failed to appear without justifiable reason is guilty of indirect
contempt which is punishable by fine and imprisonment of not
more than
A. one month
B. six months
C. one year
D. six years
29. One of the following is allowed limited practice of
medicine in our country without certificate of registration
A. postgraduate interns in hospitals
B. graduates of medicine while awaiting results of board
examination
C. physicians from other countries doing medical missions
in our country
D. a registered physician whos license is suspended

30. In which of the following instances is there a


commencement of a physician-patient relationship?
A. A patient called a physicians clinic to make an appointment.
The secretary asked what is wrong and the patient related
her complaints.

9
B. One night a patient called a physician by phone for
consultation. The physician told the patient to take
aspirin that night and see him the following day.
C. While passing by the emergency room, a physician noticed a
patient who is very pale. He asked the nurse to inform the
resident physician to see and examine the patient
immediately.
D. In a mall, a patient met her physician. She made a casual
consultation. The doctor told her to see him at his clinic a
little later.

31. Dr. Lopez is a physician in a rural health unit. One of her tasks is
to implement government program using artificial means of family
planning. Being a devout catholic, she can only prescribe natural
method. Which of the following is correct?
A. She has the prerogative to choose treatment modalities
B. She can invoke self limitation of practice based on
religious beliefs
C. She should ask to be transferred to a facility consistent with
her beliefs
D. She is liable for insubordination

32. A general practitioner referred a patient to a specialist. After


treatment, the patient paid the specialist who in turn gave a share
to the general practitioner. What is this called?
A. Fee splitting
B. Straight fee
C. Retainers fee
D. Contingent fee

34. What criminal act is considered an administrative liability of


physicians?
A. Immorality
B. Addiction to alcohol

10
C. Unethical advertisement
D. Issuance of false medical certificate

35. Mr. Santos falsified documents to get a certificate of registration


as physician. If convicted, what is the penalty?
A. Arresto menor
B. Arresto mayor
C. Prision mayor
D. Prision correccional

36. The hospital is almost full and the only vacant beds are in the
ward. An infectious case was admitted in the ward resulting to
contamination of other patients. The physician who admitted the
patient is liable under what doctrine?
A. foreseeability
B. last clear chance
C. continuing negligence
D. contributory negligence

37. Weapons, fingerprints and garments left at the crime


scene are examples of what type of evidence?
A. Real
B. Ordinary
C. Corpus delicti
D. Associative

38. What does algor mortis refer to?


A. Irritability
B. Softening of the body
C. Cooling of the body
D. Hardening of the body

39. In contrast to ante-mortem clot, post-mortem clot is


A. firm
B. soft

11
C. homogenous
D. uniform

40. A missing person is presumed dead after how many


years?
A. 4
B. 7
C. 10
D. 15

41. Infanticide is killing of a child less than what age?


A. 3 hours
B. 3 days
C. 3 months
D. 3 years

42. If no physician is available, who among the following can


sign the death certificate?
A. City fiscal
B. Judge of a trial court
C. Municipal mayor
D. Congressman

43. Physical injury is considered slight if the victim is


incapacitated for how long?
A. 6-12 hours
B. 12-24 hours
C. 1-9 days
D. 2 weeks

___D__ 44.The test used to determine whether semen is of human


origin or not is:
a. acid phosphatase
b. Berberios
c. Florence

12
d. Precipitin

___D__ 45. The crime of simple seduction is committed when the


following elements are present:
a. there must be a sexual relation between the offender and
the victim
b. The woman must be over 12 but less than 18 and single or
widow of good repute
c. The sexual act was committed by means of deceit
d. All of the above

___D__ 46. The crime committed by a man who embraced and


kissed a woman against her will is:
a. rape
b. simple seduction
c. qualified seduction
d. act of lasciviousness

___B___47.Pedophilia is a sexual perversion where a person has a


compulsive desire to:
a. engage in sexual intercourse with an animal
b. seek sexual gratification with a child of either sex
c. be near a dead body for sexual gratification
d. obtain sexual gratification by gazing at himself at the mirror

___C__ 48. The minimum gestation period compatible with viability of


the fetus according to most authors and The Philippine Law is:
a. 100 days
b. 120 days
c. 180days
d. 280days

___B___ 49. The liability of a physician who performed an abortion in


a pregnant woman upon her request but unfortunately, caused the
death of the woman is:

13
a. intentional abortion
b. intentional abortion with homicide
c. homicide
d. murder

___C___ 50. What is the clinical type of abortion when there is


hemorrhage with dilatation of the internal os and presence of
rhythmical pain?
a. missed
b. complete
c. inevitable
d. threatened

___A___ 51. The elements of intentional abortion are the following,


EXCEPT:
a. the physician commits abortion without the womans consent
b. the woman is pregnant
c. a drug was administered or an instrument was used
d. the fetus dies following the procedure

___D___ 52. Delayed untoward effects which may develop as


complications of abortion is:
a. sterility
b. fistula formation
c. pelvic adhesions
d. all of the above

__D____ 53. The medical evidence as a pre-requisite of the


prosecution to establish infanticide is:
a. that the child was born alive
b. that the child who was born alive was deliberately put to
death
c. that the child killed was less than 3 days old
d. all of the above

14
___C___ 54. The value of sex can be summarized in the following
categories, EXCEPT:
a. sex is a search of sensual pleasure and satisfaction,
releasing physical and psychic tensions
b. sex is a search for the completion of the human person
through an intimate personal union of love expressed by
bodily union
c. sex is to perpetuate ones bestiality
d. sex is a social necessity for the procreation of children

___B___ 55. Because of the bad effects of inbreeding, the law made
incestuous marriages void, and it is void and incestuous marriage
when it is:
a. marriage between stepfather and stepdaughter or
stepmother and stepson
b. marriage between collateral relatives by blood within the
fourth civil degree
c. marriage between the adopting parents and the adopted
d. marriage between the grandchildren of two brothers

___A___ 56. What is the gestational age of a fetus who is 50 cm


long; weighs 3 to 3-1/2 kg with nails beyond or at the level of the
fingertips, hair on the scalp 5 m long, and lanugo is seen only on the
shoulder:
a. at term
b. eight
c. sixth
d. seventh

___D___ 57. The confession given by a defendant accused of rape


that he had carnal intercourse with the complainant at the time and
place mentioned is:
a. involuntary confession
b. admission
c. voluntary confession

15
d. extra-judicial confession

___B___ 58. A confession is different from admission because the


latter is
a. expressed acknowledgement of the truth of his guilt as to the
crime charged
b. statement of fact which does not directly involve an
acknowledgement of guilt
c. statement of guilt
d. all of the above

___B___ 59. Physical evidence recovered from the crime scene such
as a knife with fingerprints of the suspect is called:
a. circumstantial evidence
b. direct evidence
c. tracing evidence
d. corpus delict

___D___ 60. The evidence presented which corresponds to the


precise point at issue and proves the fact in dispute without the aid of
any inference, is called:
a. hearsay
b. circumstantial
c. tracing
d. direct

___B___ 61. A statement made by a dying patient to the attending


physician in reference to the persons who inflicted injury on him is
known as:
A. privileged communication
B. dying declaration
C. hearsay evidence
D. inadmissible evidence

16
___A___ 62. The result of a lie detector test may be admissible
evidence in court if:
A. there is prior stipulation by parties concerned that the test
result will be considered admissible
B. the test is done with full and informed consent of the person
examined
C. no force, intimidation or duress is applied on the person
examined
D. the examiner is accredited as competent and reliable

63. A couple was found dead inside a car in the basement parking
area. The possible cause of death is:
A. Asphyxiation
B. Carbon monoxide poisoning
C. Anoxia
D. Carbon dioxide poisoning

64. During autopsy, which among the following should be opened


first?
A. Cranial box
B. Thoracic cavity
C. Abdominal Cavity
D. Pelvic Cavity

65. After death, which organ putrefies last?


A. liver
E. Brain
F. Lungs
G. spleen

66. What is applied on the dorsum of the hand of a person


subjected to Paraffin test?
A. Wax
B. Nitrates
C. Tattoo

17
D. Oil

67. Flattening of the skin following death is due to:


A. Cadaveric spasm
B. Rigor mortis
C. Loss of elasticity of the skin
D. Post-mortem hypostasis

68. Currens Rule is used to determine the:


A. Time of death
B. Mental status
C. Malpractice liability
D. Court procedure

69. An injury sustained from a blow inflicted by the use of a sharp


edged instrument resulting to a break in a bone is a/an:
A. Hacking wound
B. Stabbed wound
C. Avulsed wound
D. Fracture

70. Instantaneous rigidity of muscles which occurs at the moment


of death:
A. Rigor mortis
B. Cadaveric spasm
C. Death struggle
D. Cold stiffening

72. What organ is tested for chloride in drowning?


B. Brain
C. Lungs
D. Heart
E. Stomach

18
73. The performance of caesarian section when the
pregnancy is normal and uncomplicated is referred to as:
A. Defensive medicine
B. Overtreatment
C. Right to perform a service
D. Alternative treatment

74. A 34 year old female came to your clinic with chief


complaint of severe chest pain. History revealed an abrupt onset
after reading a text message from her husbands former girl
friend. Diagnostics revealed essentially normal findings, however
physical examination revealed costo-chondral tenderness which is
converted to a more serious disability or exaggeration of the real
complaint. This known as:
A. Fortuitous malingering
B. Simulated malingering
C. Fictitious malingering
D. Factitious malingering

75. What is the basis to pronounce a person dead?


A. Molecular death
B. Apparent death
C. Somatic death
D. Clinical death

76. For purpose of description, laceration of the hymen is


considered superficial when:

A. It is incomplete and does not reach the base of the


hymen.
B. It only produces congestion of the affected area.
C. When laceration does not go beyond one half the
width of the hymen.
D. When there is diffused abrasion of the affected hymen.

19
77. Before a physician carries out a physical examination of a rape
victim, he must first of all:
A. Obtain a complete history of the rape incident
B. Get the bio-data of the victim
C. Ascertain the date, time and place of the rape
D. Have a written consent from the subject or from her
guardian, if she is not of age
.
78. Strictly speaking, forensic medicine is:
A. application of law to some medical problems
B. use of medical science to shed light on some legal
problems
C. use of medicine to legal cases
D. application of law in the administration of justice

79. If a person habitually drinks alcohol, there develops a certain


degree of adaptation by the body thereby increasing the body
threshold to it. Later, an increase in percentage and quantity of
alcohol taken will be tolerated by the body, leading to diminution
of its effect. This condition is called:
A. Consumption tolerance
B. Differential alcoholic threshold
C. Constitutional tolerance
D. Alcoholic non-reflex excitability

81. A medico-legal autopsy is performed to:


A. Confirm clinical findings of a research
B. Determine if the tissue changes are due to a criminal act
C. Confirm the clinical diagnosis
D. Explain sudden death of a person apparently in good
health

82. In order to ascertain drunkenness in a driver involved in a


vehicular accident, which physical tests can one conduct:
A. Winslows test

20
B. Macdonalds test
C. Rombergs test
D. Vital capacity measurement

83. The important distinguishing presumptive characteristics


between the male and female body are as follow exceptt:
A. Hips are broader in relation to the shoulder in females; in the
male the shoulders are broader than the hips
B. The female has generally a smaller build; the male larger
C. The Adams apple in female is prominent; in male, is not
D. Female pubic hair is straight and stop short above the mons
veneris; while the male is thick, curved upwards extending up to
the umbilicus

84. In determining whether wounds were inflicted ante-mortem or


post-mortem, which distinction is not valid?
A. Ante-mortem wounds shows marks of spouting of blood from
the arteries; not found in post-mortem wounds
B. Blood is clotted in an ante-mortem wound; in post-mortem
wound is not clotted, if at all it is a soft clot
C. There is no inflammation and reparative process in ante-
mortem; but is present in post-mortem wound.
D. The edges of the ante-mortem wound gape; not so in post-
mortem wound unless caused within 1-2 hours after death.

85. A bullet that has penetrated the skin and hits bony tissue along
its course may deflect and change its direction. Such change of
trajectory is called:
A. Russian roulette
B. Ricochet of bullet
C. Bullet sporting movement
D. Missile disintegrating movement

86. Death several days after a criminal abortion is often due to:
A. pulmonary embolism

21
B. infection
C. hemorrhage
D. uterine trauma

87. Rigor mortis in the tropics lasts on the average for about how
many hours?
A. 24 hours only
B. 24 to 48 hours
C. 48 to 72 hours
D. 12 hours only

88. Underneath the skin are dense networks of fibrous and elastic
connective tissue fibers running in the same direction. Incised
wounds inflicted parallel to it will appear narrow or slit-like while
those perpendicular to it retract or gape. The pattern of fiber
arrangement is called:
A. Langers line
B. Maginot line
C. Avulsion marks
D. Diffused network

89. According to Dupuytrens classification, burns which involve the


deep fascia and muscles which may result to severe scarring or
deformity is considered to be:
A. Fourth degree burns
B. Fifth degree burns
C. Sixth degree burns
D. Seventh degree burns

90. When death by asphyxia is produced by compression of the


neck by means of ligature which is tightened by a force other than
the weight of the body is known as:
A. Death by hanging
B. Death by choking
C. Death by strangulation

22
D. Death by smothering

91. A person lost an eye after being physically assaulted. The


offender is liable for:
A. Mutilation
B. Slight physical injury
C. Less serious physical injury
D. Serious physical injury

92. Who will be held primarily liable for damages or injuries


sustained in an operation wherein the patient signed a waiver of
non claim?
A. Patient
B. Surgeon
C. Hospital
D. None of the above

93. A physician who deliberately refuses to obey a subpoena


issued by a court without any justifiable reason is guilty of:
A. Direct contempt of court
B. Criminal act of omission
C. Indirect contempt of court
D. Civil liability for damages

94. What is the liability of a physician who performed an autopsy


without proper authorization from the next of kin or competent
authorities?
A. None
B. Civil liability only
C. Criminal liability only
D. Both civil and criminal liability

95. A patient brought action against a physician for injuries caused


by the physicians employed nurse during the course of treatment.
Recovery against the physician was sought under the doctrine of:

23
A. Res ipsa loquitur
B. Borrowed servant
C. Fellow servant
D. Captain-of-the ship

96. An otorhinopharyngologist was forced by an Abu Sayyaf


member to do a caesarian operation on his wife. The physician
agreed on the condition that he will be paid PhP 1M. The physician
maybe held liable for injuries sustained under the doctrine of:
A. Foreseeability
B. Continuing negligence
C. Assumption of risk
D. Contributory negligence

97. A newborn child developed jaundice. She learned of the


physicians failure to drop silver nitrate into the childs eyes at birth,
the physician may:
A. Be liable for criminal malpractice
B. Be liable for civil malpractice
C. Be liable for criminal and civil malpractice
D. Not be liable at all

98. A physician may be presented in court as an ordinary or as an


expert witness. He is only considered as an ordinary witness:
A. When he is asked to inform the court of what he perceived
in the course of the physical examination of the patient
B. When he is asked to give opinions based on a set of facts
C. When he is asked to answer outside of the issue in
consideration of the court
D. When he is asked questions regarding his personal
qualifications as a witness

99. During an operation on a patient by a private surgeon, the


resident physician assisting him committed an error which resulted
in injury to the patient. Who is primarily liable for damages?

24
A. resident physician
B. attending surgeon
C. hospital
D. no one

100. A stab wound with serrated or zigzag borders due to alternative


thrust and withdrawing of the wounding weapon indicates that:
A. The offender had the intent to kill
B. The offender was nervous and wavering
C. The instrument is not sharp edged
D. The wound is accidental

101. The clinical records of a patient before and after discharge is


owned by:
A. Hospital
B. Patient
C. Attending physician
D. The state or government

102. Under the Sanitation Code, who is not authorized to perform an


autopsy?
A. Medical officers of law enforcement agencies
B. Health officers of local health departments
C. Medical staff members of accredited hospitals
D. Any duly licensed physician in private clinics

25
103. A male intern had sexual intercourse with a female patient with
her consent. Under what circumstance would this be considered
rape?
A. The intern promised to marry her.
B. The patient is feeble minded.
C. The patient is his wife from whom he is legally separated.
D. The patient is above 18 years of age.

104. The right of a citizen of foreign country to be allowed to be


admitted to practice medicine in the Philippines if their existing law
allowed citizens of the Philippines to be admitted to practice
medicine in their country is based on:
A. reciprocity provisions of the medical law of the Philippines
B. international law, in as much as the Philippines has adopted the
generally accepted principles of international law as part of the
law of the Philippines
C. international convention, like the World Health Organization
D. executive agreement between the heads of states

105. The doctrine which will bar a patient from recovering damages
because he assents to the risks of injury:
A. Doctrine of continuing negligence
B. Doctrine of assumption of risk
C. Contributory negligence
D. Doctrine of sole responsibility

106. Which of the following is an official oath prior to the issuance of


the Certificate of Registration?
A. Hippocratic Oath
B. Apocryphal Oath
C. Administrative Oath
D. Medical Practice Oath

26
107. An applicant in a shipping company sued the physician of a
pre-employment company for negligence due to the latters refusal
to prescribe an anti-diabetic drug for the increased blood sugar in
his FBS. Which among the following is a valid defense to be
invoked by the physician?
A. Right to choose patients
B. Right to limit practice
C. Absence of physician-patient relationship
D. Absence of consideration

108. A 35 year old lingerie model came to your clinic with chief
complaint of foul smelling vaginal discharge. Examination revealed
gram negative diplococci. She told you that the treatment should
be kept in strict confidentiality. Furthermore, she revealed that she
got her illness from her promiscuous lover. She placed rat poison
in his lovers tomato juice before she left to see you. You promised
that you will keep it between the two of you. The following
morning, you learned that your patients lover died of internal
hemorrhage which can be attributed to a rat poison. What will you
do if summoned to testify on the death of your patients lover?
A. I will raise the defense of a privileged communication.
B. I will raise the right of the patient for privacy.
C. I will raise the issue of my right to limit practice, not in a medico-
legal practice.
D. I will testify.

109. This technique of interrogation is applicable when there are two


or more persons who allegedly participated in the commission f a
crime:
A. emotional appeal C. bluff on split-pair technique
B. Mutt and Jeff technique D. stern approach
1987 p. 35

27
110. Broken headlights glass found at the crime scene in hit and run
homicidematbe associated with the car found in the repair shop
is an example of:
A. Corpus delicti evidence C. associative evidence
B. tracing evidence D. deposition
1987 p. 16

111. A branch or division of law which deals with the rules


concerning pleadings, practices and procedures in all courts of
the Philippines:
A. Remedial law C. Criminal law
B. Civil law D. Speccial law
1987 p. 5

112. This is composed of laws which are produced by the countrys


legislation and which are defined, codified and incorporated
by the law-making body:
A. Commonlaw C. Customary law
B. Statutory law D. Case law
1987 p. 3

113. In determining the sex of the skeleton, the following bones must
be studied except:
A. sternum B. pelvis C. femur D. mandible
p. 80

114. This test is positive to any substance containing hemoglobin:


A. Takayama test C. Hemin crystals test
B. Acetone-haemin of Wagenhaar test D. Teichmans blood
crystals
p. 99

115. The following are characteristics of medico-legal autopsy


except:

28
A. consent of relatives are not needed
B. correlates tissue changes to the criminal act
C. notation of all abnormal findings
D. must be specific for the purpose of determining whether it is
in relation to the
criminal act

116. Which of the following is not a characteristic of an exit wound:


A. edges are everted C. paraffin test maybe
positive
B. does not manifest any definite shape D. tattooing is always
absent
p. 362-363

117. Which of the following is a test for the presence of powder


residues on the clothings:
A. Gonzales test C. paraffin test
B. Walkers test D. Lungs test
p. 380

118. Parents of this type often have personality disorders and are
coolly indifferent to the destructive nature of their action:
A. intermittent child abuser C. constant child
abuser
B. one-time child abuser D. ignorant abuser
P. 480

119. Merely a corroborative proof of live birth:


A. presence of heart sound C. diaphragm reaches the level of
6 or 7th ribs
th

B. restlessness and crying D. stomach contains mucous, air


bubbles and saliva
p. 577

120. Which of the following are legitimate children:

29
A. adopted children C. adulterous children
B. natural children D. spurious children
P. 587

121. Causes of stillbirth:


A. immaturity C. accidents in the delivery
B. congenital malformation D. all of the above
P. 576

122. The following are appearance of the lungs if respiration takes


place after birth except:
A. lungs fill the thoracic cavity and overlapping the heart and
thymus gland
B. when squeezed between the finger and thumb they do not
crepitate
C. surface is covered with mosaic of expanded air vesicles,
giving a marble appearance
D. lungs are voluminous, with rounded edges and pink mottled
color
p. 579

123. It concerns the state in its sovereign or political capacity to


enforce or regulate the exercise of a right, duty or obligation
impose on the subject:
A. criminal law C. law on torts and damages
B. law on obligation and contracts D. private law
p. 6

124. Criminal acts as ground for reprimand, suspension or


revocation of registration certificate:
A. insanity
B. addiction to alcoholic beverages
C. knowingly issuing any false medical certificate
D. gross negligence in the practice of his profession
p. 200

30
125. Which of the following is not a duty of a physician with regards
to dying declaration?
A. He must observe carefully the mental condition of the
patient.
B. He must avoid leading questions.
C. He must immediately put down in writing the patients
statement.
D. He must produce the statement taken and decide by
himself the relevant points and its admissibility.
p. 452
126. Which of the following conditions must a medical witness
comply with a subpoena?
A. When the court issuing the subpoena has no jurisdiction
over the subject matter and person of the case.
B. When a physician is attending to an emergency and no
one is available and competent enough to be his substitute.
C. When the place of residence is 30 kilometers or less from
the court issuing the subpoena.
D. When the physician is suffering from an infirmity which
may be aggravated if ever he complies with the subpoena.
p. 461

31
127. Which of the following is not a requisite of the privileged
information between the physician and his patient?
A. That the information is confidential and if disclosed, will
tend to blacken the character of the patient
B. That the person against whom the privilege is claimed is
one duly authorized to practice medicine, surgery or
obstetrics
C. That the privilege is claimed in a criminal case not in any
civil proceedings
D. That the physician acquired the information while he was
attending the patient in his professional capacity
p. 443

128. Which of the following is an administrative liabilities of a


hospital?
A. improperly administering a hypodermic injection
B. keeping a hot water bottle too long on a patients body
C. administering the wrong blood to the right patient
D. failing to place sideboards on a bed after a nurse decided
they were
necessary
p. 318

129. Which is a qualitative overtreatment of physician?


A. indiscriminately prescribing medicine having similar effects
B. hospitalization of patient for a minor ailment
C. excessive testing
D. longer period of treatment
p. 297

130. An expert medical witness has the following qualification:


A. duly licensed medical practitioner
B. duly licensed medical practitioner with at least 5 years of
active

32
practice
C. duly licensed medical practitioner who has specialized in a
particular branch of medicine
D. duly licensed medical practitioner who is a specialist in the
line
along which he is testifying
p. 453

131. Ghost surgery is committed when:


A. the physician who performs the operation is not the
physician to
whom the patient gave his consent for the operation
B. the physician performs an operation beyond that which has
been
consented by the patient
C. an operation is said to have been performed when in fact
nothing
was done
D. an operation is performed other than that to which the
patient gives
consent
p. 389

132. Failure to seek further medical assistance if symptoms persists:


A.Doctrine of continuing negligence C. Doctrine of last clear
chance
B. Doctrine of foreseeability D. Doctrine of contributory
negligence
p. 248

133. Contributory negligence cannot be applied when there is:


A. heavy sedation B. old age C. semiconsciousness D. all
of the above
p. 250

33
134. Operating without consent constitute an assault, and an
assault, even without specific proof of injury resulting from it,
entitles the plaintiff to this damages:
A. liquidated damages C. temperate danagtes
B. nominal damages D. moderate damages
p. 374

135. An administrative complaint was filed with the PRC against a


physician who was eventually found guilty. This may result in:
A. Reprimand, suspension or revocation of license to practice
B. Imprisonment for criminal negligence
C. Civil damages paid to the patient to compensate for injuries
sustained
D. Need to be re-certified

Ans: A

136. A gynecologist performed a total abdominal hysterectomy on a


patient with myoma uteri. Before closing the abdomen, he
discovered that the patient had cancer of the cecum so he
referred the patient to a general surgeon to perform the
resection. Three days post-op, the patient had to be reopened
and a piece of gauze was found. Who may be held liable under
the Captain of the ship Doctrine?
a. The operating room nurse
b. The second surgeon
c. The gynecologist
d. The anesthesiologist

Ans: C

137. Dr. Ola refused to render service in an emergency case in an


NPA controlled territory. Which is correct?
a. His license will be revoked

34
b. He will be civilly liable for the injuries sustained by the
patient by the patient for his failure to treat.
c. He can refuse service if there is risk on his life.
d. He violated his professional oath.

Ans: C

138. As a result of the deformity he sustained from a facelift


operation, Mary Juana sued Dr. Hanny Bal for mental anguish,
social humiliation and moral shock. The type of damages she
can recover as a result of these is;
a. Actual damages
b. Compensatory damages
c. Exemplary damages
d. Moral damages

Ans: D

139. A patient suffering from a painful incurable disease asked his


physician to give him lethal dose of poison. OUT OF PITY, the
physician gave the poison which the patients swallowed and a
result died. The physician is guilty of:
a. Murder
b. Homicide
c. Giving assistance to suicide
d. Dishonorable conduct

Ans: A

140. There is negligence in the following circumstance


a. Not giving antibiotics in cataract extraction
b. Not giving anti-rabies vaccine to a patient bitten by a dog
c. Not giving tetanus toxoid to a patient who stepped on a
nail
d. Not doing a skin test before administering IV oxytocin

35
Ans: C

141. Originally there were railings on the bed of the patient. Mr. A, the
father removed the railings so that he himself can sleep on the
bed. His child fell with a broken skull. What is the doctrine
applicable?
a. Res ipsa loquitor
b. Borrowed servant doctrine
c. Contributory negligence
d. Common knowledge

Ans: C

142. When a body is the subject matter of a criminal investigation, it


may exhumed:
A. Anytime
B. 3 years after burial
C. 5 years after burial
D. 10 years after burial

Ans: A

143. This type of injury need not be reported to police authorities


under PD 169:
A. Serious physical injuries
B. Mutilation
C. Slight physical injuries
D. Less serious physical injuries

Ans: C

144. Carnal knowledge means


A. Knowledge about the nature of sexual life
B. Rupture of the hymen

36
C. Slightest penetration of the female sexual organ by the
male sex organ
D. Full penetration is necessary

Ans: C

145. Bumper fracture of the leg bones in pedestrian vehicle


collision is a consequence of:
A. Secondary impact
B. Hit and run injuries
C. Primary impact
D. Run over injuries

Ans: C

146. This finding in death is not consistent and may or may not
appear on a person ho died:
A. Rigor Mortis c. Putrefaction
B. Algor Mortis d. Cadaveric spasm
Ans: D

147. This finding suggests suicide:


a. Indication of struggle
b. Presence of hesitation cuts or tentative incisions
c. Multiple wounds in different parts of the body
d. No history of depression

Ans: B

148. Disorientation, mental confusion, dizziness, sensory


disturbances exaggerated emotional state suggests that the
blood alcohol level is:
A. 150-300 mg%
B. 400 mg%
C. 100 mg%

37
D. 20 mg%

Ans: A

149. The prominence and coloration of superficial veins more


visible among people with fair complexion during
decomposition is called:
Skin Arborization
Venous
Marbolization
Venous visibility

Ans: C

150. Tardieu spots are associated with:


Hanging
Manual strangulation
Ligature strangulation
Choking

Ans: A

151. The crime of mutilation is understood to mean:


inflicting physical injury
Rendering any part of the body useless
Intentionally cutting off any reproductive part of the
body
Accidentally depriving

Ans: C

152. Fingerprinting is considered to be the best valuable


method of identification. It is universally used because
fingerprints, until decomposition, are not changeable from

38
the time they are formed in the fetus during pregnancy in
the:
Second month
Third month
Fourth week
Fifth month

Ans: C

39
153. A child born out of lawful wedlock where the parent
eventually had a valid marriage is considered:
Incestuous
Sacrilegious
Adulterous
Legitimated

Ans: D

154. Physical injuries which develop on parts of the body when


bone tissues are deep seated caused by forcible impact of
hard blunt objects are usually:
Abrasion
Contusion
Hematoma
Lacerated wound

Ans: B

155. Evidence obtained regarding the issue of the truth serum


is not admissible in court because:
The information is obtained under duress
The information given by the suspect is not
voluntary since he is under the influence of a
drug
The truth serum is still in experimental stage
The information is obtained through deceit

Ans: C

156. Algor Mortis refers to:


Irritability
Softening of the body

40
Cooling of the body
Hardening of the body

Ans: C

157. When blood accumulates in the most dependent portions


of the dead body, it is called
Primary flaccidity
Secondary flaccidity
Post-mortem lividity
Algor mortis

Ans: C

158. The following manner of death should be autopsied,


EXCEPT;
suicides
death unattended by physician
accidental death
death occurring in natural manner

Ans: D

159. Its main function is the transformation of mechanical


energy by the hit of the firing pin on the percussion cap to
chemical energy:
Primer
Powder
Bullet
Projectile

Ans: A

160. When the color of the contusion changes to green, the


contusions estimated age is

41
2-4 days
4-5 days
7-10 days
14-15 days

Ans: B

161. Cutis Galina or Washer womans hands and feet on the


corpse signifies that:
The body has been in water for sometime
The body is that of a laundry woman
The body died for drowning
The body died for syphilis

Ans: A

162. The presence of weapon which is highly grasped by the


hand of a victim of a shooting incident is an example of
Rigor mortis
Death stiffening
Cadaveric spasm
Cadaveric rigidity

Ans: C

163. The most prominent sign of death is:


Progressive fall of the body temperature
Cessation of heart action and circulation
Cessation of respiration
Insensibility of body loss of power to move

Ans: A

___C__ 164. The crime of qualified seduction can be committed on a


woman-victim above the age of 18 when:

42
a. the sexual act was done deceitfully
b. the sexual act was committed with force or intimidation
c. the offender is the father or brother of the victim
d. the woman is a prostitute

__C___ 165.The liability of a school teacher enticing a16 year old girl
who is a virgin into having sexual intercourse with him is:
a. statutory rape
b. simple seduction
c. qualified seduction
d. acts of lasciviousness

___C__ 166. A physician who fondled the breast of his female patient
during physical examination may be charged with:
a. malpractice
b. qualified seduction
c. act of lasciviousness
d. moral turpitude

___A__ 167.The right of a husband or wife to have the right of


society, service, companionship and sexual relation with the spouse
is called:
a. right of consortium
b. husbands right
c. right of marriage
d. wifes right

___D__ 168.Which of the following marriages shall be considered


void from the beginning?
a. between the adopting father or mother and the adopted
b. between the legitimate children of the adoptor and the
adopted
c. between stepfather and stepdaughter
d. all of the above

43
___A__ 169.A form of sexual deviation whereby pain is the principal
factor for satisfaction, either as a recipient or inflictor is called:
a. algolagnia
b. irrumation
c. buggery
d. bestiality

___C___170. A sex deviant who submits to surgery to change his or


her anatomical sex is known as:
a. homosexual
b. intersexual
c. transsexual
d. transvestite

___B___171.A woman who was treated in the emergency room for


bleeding from several lashes on her back incurred by her
lover while having intercourse is a victim of:
a. fetishism
b. sadism
c. uranism
d. masochism

___D__172. The abnormal sexual act committed by a farmer who


had sexual relation with his carabao:
a. masturbation
b. homosexuality
c. pedophilia
d. zoophilia

___B___ 173.A test to prove live-birth is when the middle ear of a


child, before birth ,is filled with gelatinous, embryonic connective
tissues but disappears after birth is:
a. Foderes
b. Wredins
c. Wnslos

44
d. Breslaus

___D___174.One of the test used to determine life before death of a


fetus is to float the lungs in water is:
a. Wrendins test
b. Icards test
c. Breslaus test
d. Hydrostatic test

___A___175.Thefollowing are signs of intra- uterine fetal death,


EXCEPT:
a. HCG test positive
b. Absence of fetal movement
c. Ultrasound evidence of cranial bones overlapping
d. No progress in the enlargement of the uterus

45
___D___ 176. The punishment for simulation of births and the
substitution of one child for another performed by a physician is:
a. fine of not less than P1000
b. prison mayor and a fine of not less than P1000
c. Prison mayor
d. Prison mayor, a fine of not less than P1000 and temporary
special disqualification

___A___ 177. Which of the following types of abortion is criminal and


punishable by law?
a. intentional
b. post-mortem
c. spontaneous
d. therapeutic

___B___ 178. Which of the following may be considered justifiable


ground for therapeutic abortion?
a. German measles
b. Congestive heart failure
c. Minimal pulmonary tuberculosis
d. Hypertension

___D___ 179. A woman who shall practice an abortion on herself or


shall consent that any other person should do so shall suffer the
penalty of:
a. prison mayor
b. arresto mayor
c. prison correctional at its minimum and medium period
d. prison correctional at its medium and maximum period

___C___ 180. Infanticide is the killing of a child less than:


a. 3 months
b. 3 weeks
c. 3 days

46
d. 3 years

____B__ 181. The crime committed by Mr. Aguilar who killed their
baby a day after her wife delivered suspecting that his wife was
having an affair with her officemate is:
a. murder
b. parricide
c. homicide
d. infanticide

___A___ 182. For a dying declaration to be admissible in court which


statement is NOT valid?
a. the declarant was not the victim
b. the declaration must be with regards to the impending death
c. the declarant was conscious of his impending death
d. the declarant was in full possession of his mental faculties
when he made the declaration

183. If a physician is convicted of an administrative offense, he can


make an appeal within how many days after receipt of the
promulgation?
A. 7
B. 14
C. 30
D. 60

184. What is the minimum qualification of a physician to be accepted as


expert witness?
A. license to practice medicine
B. ten years of medical practice
C. at least 21 years old
D. a certified specialist

185. Any person who performs any act constituting practice of


medicine but is not qualified to do so is

47
A. committing malpractice
B. committing fraud
C. acting under extraordinary circumstance
D. illegally practicing medicine

186. Mang Ambo utilizes light and water in his healing sessions.
What is he?
A. naprapathist
B. neuropathist
C. naturopathist
D. masseuse

187. A fiduciary physician-patient relationship is based on


A. law
B. court order
C. mutual consent
D. mutual trust

188. A patient who had an eclamptic seizure is in coma and is being


treated by a team of physicians. What is the pattern of relationship
between the physicians and the patient?
A. active-passive
B. guidance-cooperation
C. mutual participation
D. consensual
189. In a physician-patient relationship, what is expected of the
physician?
A. avoid errors of judgement
B. possess average knowledge and skills
C. be a specialist in a certain field
D. act like an authority in the field

190. The physician-patient relationship ends when


A. the patient has paid the physician
B. the physician has left for a convention abroad

48
C. the period specified in the contract has expired
D. the patient says he can no longer afford medical care

191. A physician may choose patients whom to serve but he may not
do so in
A. places where there are no doctors
B. situations when they have no valid reason
C. cases of emergency
D. he is the patients choice

B 192. Blood that does not clot, acidic in pH, with the presence
of epithelial cells and Doderlain microorganisms is
probably:
A. Blood from a hemophiliac
B. Menstrual blood
C. Blood from a person with metabolic acidosis
D. Septic blood

Chapter IIIMedico Legal Aspects of Identification; p.104


Legal Medicine by Pedro Solis (1987 ed)

B 193. In a cadaver, the chemical change of body fat which


takes place in wet conditions and results in a soft brownish
white substance is known as:
A. Mummification
B. Adipocere formation
C. Bacterial Putrefaction
D. Lytic changes
Chapter IVMedico-Legal Aspect of Death; p.149
Legal Medicine by Pedro Solis (1987 ed)

C 194. Dr. Pedro, the MHO of Aurora Municipio shall perform an


autopsy on a dead body when:

49
A. Requested by any relative of the victim
B. Verbally directed by the police
C. Upon order of the mayor
D. Ordered by the Provincial Hospital Medical Director

Chapter VMedico Legal Investigation of Death; p.165


Legal Medicine by Pedro Solis (1987 ed)

A 195. Head injury that may be found in areas of the skull


offering the least resistance is known as:
A. Locus minoris resitencia
B. Countre-coup injuries
C. Remote injuries
D. Coup injuries

Chapter XI-Injuries in Different Parts of the Body; p.301


Legal Medicine by Pedro Solis (1987 ed)

C 196. Petra scratched Juana in the face in an altercation. The


doctor attended the injury that resolved in less than 10
days but a keloid scar formed. The legal classification of
this injury is:
A. Mutilation
B. Slight injuries
C. Serious physical injuries
D. Less serious injury

Chapter IXMedico Legal Aspects of Physical Injuries;


p.250
Legal Medicine by Pedro Solis (1987 ed)

B 197. Sexual intercourse is not an element of this crime:


A. Seduction
B. Abduction

50
C. Concubinage
D. Adultery

Chapter XXI-Medico-Legal Aspects of Sex Crimes; p.520-


521
Legal Medicine by Pedro Solis (1987 ed)

D 198. This will not cause hymenal laceration:


A. Riding horseback
B. Medical instrumentation
C. Carnal knowledge
D. Interlabial sex

Chapter XXI-Medico-Legal Aspects of Sex Crimes; p.492


Legal Medicine by Pedro Solis (1987 ed)

D 199. This type of prostitute works regularly in a prostitution


den under the direct supervision of a mama-san.
A. Hooker
B. Call girl
C. Pick up
D. Factory girl

Chapter XXI-Medico-Legal Aspects of Sex Crimes; p.524


Legal Medicine by Pedro Solis (1987 ed)

C 200. The crime of infanticide is almost always committed in


the:
A. Nursery
B. Delivery room
C. Home
D. Maternity clinic

Chapter XXVI-Infanticide; p.584


Legal Medicine by Pedro Solis (1987 ed)

51
B 201. Pedro is impotent yet her wife gave birth to a healthy
baby while in lawful wedlock. Which is true?
A. The childs legitimacy cannot be questioned
B. Proof of impotence will overthrow presumption of
legitimacy
C. The child upon birth can be declared illegitimate
D. None of the above

Chapter XXX-Impotency and Sterility; p.620


Legal Medicine by Pedro Solis (1987 ed)

E 202. What causes sexual dissatisfaction after marriage?


B. Dyspareunia
C. Venereal disease
D. Frigidity
E. All of the above

Chapter XXIX-Medico-Legal Aspects of Marital Union and


Dissolution; p.615
Legal Medicine by Pedro Solis (1987 ed)

D 203. Jose developed a massive stroke two days after


contracting marriage with Josefa. Thereafter, he became
physically incapable of assuming the obligations of marriage
and the condition appears to be irreversible. Which is applicable
in this case?
A. The marriage is null and void.
B. There is legal basis of annulling the marriage.
C. Josefa can re-marry if after a period of time Jose does
not recover.
D. The marriage is valid.

Chapter XXIX-Medico-Legal Aspects of Marital Union and


Dissolution; p.618

52
Legal Medicine by Pedro Solis (1987 ed)

C 204. Dra. Petras homosexual husband left her for another man.
Out of frustration, she had a one-night stand with a man she met in a
bar. Her husband filed a complaint against her. She can be held
liable for:
A. Infidelity
B. Concubinage
C. Adultery
D. Marital impropriety

Chapter XXIX-Medico-Legal Aspects of Marital Union and


Dissolution; p.618
Legal Medicine by Pedro Solis (1987 ed)

53
B 205. Jose was found guilty of large-scale estafa and was to be
imprisoned. He suddenly became insane. Which should take
place?
A. He should be sent to prison.
B. His imprisonment should be deferred until he
recovers.
C. He should undergo another trial upon recovery of
sanity
D. He should be pardoned.

Chapter XXXII-Disturbance of Mentality; p.627


Legal Medicine by Pedro Solis (1987 ed)

B 206. This is an organic poison:


A. Codeine
B. Amyl alcohol
C. Methamphetamine
D. Phenobarbital

Chapter XXXIV-Medico-Legal Aspect of Poisoning; p.727


Legal Medicine by Pedro Solis (1987 ed)

B 207. Cutaneous stains in areas where the corrosive liquid


has been spilled is seen in this type of chemical poisoning due
to:
A. Nitric Acid
B. Sulfuric Acid
C. Hydrochloric Acid
D. Citric Acid

Chapter XXXIV-Medico-Legal Aspect of Poisoning; p.732


Legal Medicine by Pedro Solis (1987 ed)

54
B 208. Jose presents with gouty tophi and arthritis.
Examination of the joints showed simple effusion. Jose works
as a painter in the shipyard. He is probably suffering from:
A. Hydrocyanic poisoning
B. Chronic lead poisoning
C. Rat poisoning
D. Drug poisoning
Chapter XXXIV-Medico-Legal Aspect of Poisoning; p.728
Legal Medicine by Pedro Solis (1987 ed)

A 209. Glucose is administered orally or intravenously in


poisoning to:
A. Restore depleted glycogen
B. Eliminate the poison
C. Neutralize the poison
D. Biotransform the poison to something inert

Chapter XXXIV-Medico-Legal Aspect of Poisoning; p.721


Legal Medicine by Pedro Solis (1987 ed)

C. 210. Specific supervision, control and regulation of medical


practice is a function of:
A. Professional Regulatory Commission

B. Board of Medical Education


C. Board of Medicine
D. Dept. of Health
Chapter II - Licensure and Regulatory Laws; p.8
Medical Jurisprudence by Pedro Solis (1988 ed)
B 211. The non-issuance of a Certificate of Registration to a
successful examinee may be due to one of the following:
A. In trial for a criminal offense involving moral turpitude
B. Found guilty by the Board of Medicine of immoral or
dishonorable conduct
C. Appears to be of an unsound mind

55
D. All of the above
Chapter II - Licensure and Regulatory Laws; p.35
Medical Jurisprudence by Pedro Solis (1988 ed)

B 212. This act does not constitute the practice of medicine:


A. Physical examination of the patient
B. Collection of professional fees
C. Diagnosis and treatment
D. Prescribing drugs

Chapter III The Practice of Medicine; p.39


Medical Jurisprudence by Pedro Solis (1988 ed)

B. 213. Beautician Vicky treats acne and dishes out topical


lotions to clients. Which of the following is correct?
A. Becky is performing a cosmetic function
B. Becky is illegally practicing medicine
C. Becky is administratively liable under PRC rules
D. Beckys parlor should be padlocked by the police
authorities

Chapter III The Practice of Medicine; p.48


Medical Jurisprudence by Pedro Solis (1988 ed)

B. 214. Dr. Jose left a forceps in the abdominal cavity after an


operation. The applicable legal principle in this case is:
A. Doctrine of vicarious liability
B. Doctrine of res ipsa loquitur
C. Doctrine of contributory negligence
D. Doctrine of forseeability

Chapter X Doctrines Applied in Malpractice Cases;


pp.239-248
Medical Jurisprudence by Pedro Solis (1988 ed)

56
D 215. Securing the patients consent for treatment is an
established standard of practice. Failure to secure it is
malpractice. To be valid, consent must be:
A. Informed and enlightened
B. Obtained voluntarily
C. Subject matter must be legal
D. All of the above

Chapter VI Rights of Patients; pp.125-126


Medical Jurisprudence by Pedro Solis (1988 ed)

D 216. A patient-physician patient relationship exists here:


A. Insurance eligibility examination
B. Pre-employment examination to determine fitness for
work
C. Determination of sanity by a court appointed physician
D. Attending physician refers the patient to another doctor

Chapter IV Physician-Patient Relationship; pp.69-71


Medical Jurisprudence by Pedro Solis (1988 ed)

D 217. This type of service cannot be offered in a patient-


physician relationship:
A. Emergency care and treatment
B. Examination, diagnosis, and treatment of particular
problems
C. Examination, diagnosis and treatment of all medical
problems
D. Therapeutic abortion services

Chapter IV Physician-Patient Relationship; p.77


Medical Jurisprudence by Pedro Solis (1988 ed)

57
D 218. The patient-physician relationship does not contemplate
one of the ff. promises or guaranties:
A. Treatment will be beneficial
B. Treatment will not be injurious to the patient
C. Treatment will result in a definite outcome
D. Treatment will be gratuitous

Chapter IV Physician-Patient Relationship; p.75


Medical Jurisprudence by Pedro Solis (1988 ed)
B 219. Burden of evidence needed to convict a physician for
criminal malpractice:
A. Substantial evidence
B. Proof beyond reasonable doubt
C. Preponderance of evidence
D. Circumstantial evidence

Chapter VIII Criminal Liabilities of Physicians; p.199


Medical Jurisprudence by Pedro Solis (1988 ed)

A 220. Dr. Pogis wife filed an administrative complaint before


the PRC against her husband who was living with
a mistress for having abandoned her and her three
children. The doctors immoral acts are grounds for
revoking his registration certificate and classified as:
A. Personal disqualification
B. Unprofessional conduct
C. Unethical conduct
D. Criminal act

Chapter XVI Liabilities of Physicians; p.179


Medical Jurisprudence by Pedro Solis (1988 ed)

A 221. An OB-Gyne resident saw a patient in the ER


complaining of left sided abdominal pain. She was given
celecoxib and sent home. She came back in shock for

58
ruptured ectopic pregnancy. The hospital is liable under
the doctrine of:
A. Vicarious liability
B. Independent contractor
C. Borrowed servant
D. Corporate liability

Chapter XIII- Liabilities of Hospitals pp.327-328


Medical Jurisprudence by Pedro Solis (1988 ed)

59
B 222. Which of the following statements is valid:
A. Hospital resident physicians have the option to do private
practice in the hospital where they are training
B. Private hospitals have the right to screen and choose the
medical staff that may be allowed to practice in the hospital
C. Any licensed physician can bring his patient to any
government hospital and treat them there
D. Postgraduate interns are liable for their errors while
rendering medical care in the hospital

Chapter XIII- Liabilities of Hospitals p.311


Medical Jurisprudence by Pedro Solis (1988 ed)

D 223. For purposes of establishing vicarious liability, one of


the following is generally not regarded as a hospital
employee:
A. Resident physicians
B. Staff physicians
C. Chief of Clinics
D. Post-graduate interns

Chapter XIII- Liabilities of Hospitals p.327


Medical Jurisprudence by Pedro Solis (1988 ed)

C 224. Which cannot be sued and thus vicarious liability does


not apply:
A. Government hospitals with proprietary functions
B. Charity hospitals
C. Public hospitals with governmental functions
D. Private hospitals

Chapter XIII- Liabilities of Hospitals p.314


Medical Jurisprudence by Pedro Solis (1988 ed)

60
B 225. In a malpractice case, Dr. Procopio was ordered by the
court to produce at trial the patients charts, laboratory
and x-ray results and other documents under his
possession and control. The process served on him to do
this is called:
A. Ordinary subpoena
B. Subpena duces tecum
C. Subpoena ad testificandum
D. Court summons

Chapter XXI Medical Witness and the Court; pp.456-457


Medical Jurisprudence by Pedro Solis (1988 ed)

B 226. A physician treated a terrorist for gunshot injuries. In the


course of treatment, the patient told the doctor he killed
and beheaded hostages. When the terrorist was captured,
the physician was called to testify. What should he do?
A. Keep his mouth shut as the information is privileged
B. Testify against the terrorist
A. Invoke patient confidentiality
C. Refuse to testify

Chapter XXI Medical Witness and the Court; pp.443-447


Medical Jurisprudence by Pedro Solis (1988 ed)

C 227. A physician must appear in response to a subpoena if his


place of residence is how many kilometers from the court
issuing the subpoena?
A. 100 kilometers
B. 50 kilometers
C. Less than 50 kilometers
D. Must appear regardless of distance

Chapter XXI Medical Witness and the Court; pp.461

61
Medical Jurisprudence by Pedro Solis (1988 ed)

B 228. A family physician has a patient with a severe


cardiovascular disease. What should be the most
appropriate thing for him to do?
A. Refer the patient to another family physician who is
more experienced
B. Refer the patient to a physician whose specialty covers
the case
C. Refer the patient to a more senior consultant
D. Exhaust all means to provide a correct diagnosis for
proper management of the case

Code of Ethics of the Medical Profession in the


Philippines; p.476
Medical Jurisprudence by Pedro Solis (1988 ed)

C 229. A patient was dissatisfied with the treatment provided by


his Dr. Jose. He transferred to another hospital to be
treated by Dr. Juan. Dr Jose practices also in that hospital
and filed a case against Dr. Juan. Choose the best
answer.
A. Juan should return the patient to Joses
management.
B. Juans action violates code of ethics as relates to
another physician.
C. Juans action if done in good faith is ethical
D. Jose should be admonished by the hospital
authorities.

Code of Ethics of the Medical Profession in the


Philippines; p.481
Medical Jurisprudence by Pedro Solis (1988 ed)

62
C 230. A 27-year old woman who has previously received no
prenatal care presents with spotting at term but not yet in
labor. On ultrasound she has a placenta previa, but she
refuses cesarean section for any reason. What should the
physician do?
A. Operate since this is an emergency
B. Respect the patients wishes
C. Refer the matter to the ethics committee as this involves
the life of the mother and the fetus
D. The physician should get a court order to force a
cesarean section

Code of Ethics of the Medical Profession in the


Philippines; pp.475-476
Medical Jurisprudence by Pedro Solis (1988 ed)

___C__ 231.A sexual deviation characterized by an obsessive desire


to wear the attire of the opposite sex is:
a. transsexuality
b. homosexuality
c. transvestism
d. hermaphoditism

___D____232.A woman who develop menopause at 20 is in a state


of:
a. senility
b. premature aging
c. menarche
d. climacterium praecox
___B___ 233. The operation done on a dead pregnant woman is:
a. laparotomy
b. post mortem Cesarean section
c. classical Cesarean section
d. primary Cesarean section

63
___D___ 234. Evidence not obtained from the personal knowledge or
experience of the witness but derived from what he has
heard or read about is called:
a. expert evidence
b. privileged communication
c. associative evidence
d. hearsay evidence

___A___ 235. The form of evidence given by the attending physician


of a dying patient who related to him (Doctor) that his
compadre killed his wife is:
a. hearsay evidence
b. dying declaration
c. testimonial evidence
d. deposition

236. A patients condition requires surgery but she refuses to submit


to such procedure. What should the physician do?
A. Seek another doctors opinion
B. Politely withdraw as attending physician
C. Ask other members of the family to give consent
D. Simply abandon the patient

237. A professional fee was agreed upon by the physician and patient
before the start of treatment. What kind of professional fee is it?
A. Retainer
B. Contingent
C. Straight
D. Simple contractual

238. In which of the following situations is medical practice limited by


other sources and not self-imposed?
A. A physician charges large amount of fee resulting to
limited patients
B. A physician limits his practice to certain days of the week

64
C. A physician decides to practice only in the hospital
D. A physician limits his practice because of dictates of
conscience

239. Which of the following statements is true?


A. Any graduate of medicine can automatically practice in
any hospital
B. A physician can practice in hospitals once he becomes a
specialist
C. A hospital is obliged to give all licensed physician the
privilege to practice
D. A hospital has the discretion to choose physicians
whom to give the privilege to practice

240. In which of the following situations is consent not necessary?


A. There is no one around to sign the consent
B. The patient is unconscious
C. The case is emergency
D. The procedure is known to be safe

241. A patient diagnosed to have cervical cancer should be informed


of the diagnosis because of her right to
A. confidentiality
B. treatment
C. give consent
D. disclosure

242. Manny, a 19 year old married was diagnosed to have acute


appendicitis. If all of the following are available, who should sign the
consent?
A. Manny
B. The spouse
C. His father
D. His mother

65
243. Confidentiality of information does not apply in which situation?
A. Patient waives her right
B. The case is interesting
C. The condition is a rare case
D. The patient has no relative around

244. A patient consulted at a physicians clinic. He related his


symptoms and the doctor gives treatment. What type of consent was
used?
A. Implied
B. Oral
C. General
D. No consent

245. Mr. Santos falsified documents to get a certificate of registration


as physician. If convicted, what is the penalty?
A. Arresto menor
B. Arresto mayor
C. Prision mayor
D. Prision correccional

246. A physician failed to report treatment of physical injuries to


proper authority. What is the duration of imprisonment if found guilty?
A. 6 months to one year
B. 1 to 3 years
C. 3 to 6 years
D. 6 to 12 years

247. Being a graduate of Medicine is what type of circumstance to a


criminal liability?
A. Exempting
B. Aggravating
C. Mitigating
D. Alternative

66
248. In which of the following circumstances is there definite
negligence?
A. Not giving antibiotics in cataract extraction
B. Not giving anti-rabies vaccine to a patient bitten by a dog
C. Not giving tetanus toxoid to a patient who stepped on
a nail on the ground
D. Not doing skin test before administering IV oxytocin

249. The doctrine of res ipsa loquitur applies to which of the following
situations?
A. Plastic surgery to remove a scar resulted to an uglier scar
B. Diagnosis of appendicitis which turned out to be
salpingitis
C. Doing craniotomy in a dying victim of vehicular accident
D. Burns during application of perineal heat after
delivery

250. General compensatory damages include which of the following?


A. Hospital expenses
B. Funeral expenses
C. Loss of earning capacity
D. Drugs brought from outside pharmacy

250. The drug used as truth serum in detecting deception is


A. meperidine
B. hyoscine hydrobromide
C. sodium amytal
D. sodium pentothal

251. What characteristic of a living person can easily be changed?


A. Mental memory
B. Degree of nutrition
C. Grade of profession
D. Complexion

67
252. What is the gait of a person with tabes dorsalis
A. Ataxic
B. Paretic
C. Spastic
D. Wadding

253. Which of the following approximates a persons height?


A. Ten times the length of the head
B. Three times the distance from vertex of skull to symphysis
pubis
C. Distance from tips of middle fingers with arms
extended
D. Two times the distance between supra-sternal notch and
symphysis pubis

254. What is the most valuable method of identification?


A. Handwriting
B. Fingerprinting
C. Dental identification
D. Identification of skeleton

255. What is the art and study of recording fingerprints for


identification?
A. Dactyloscopy
B. Dactylography
C. Poroscopy
D. Graphology

256. Diaphanus testwas used to determine peripheral circulation.


How is this done?
A. Place ligature around the base of the finger
B. Spread fingers wide and view through strong light
C. Subcutaneous injection of fluorescein
D. Apply heated material to the skin

68
257. What is post-mortem caloricity due to?
A. Temperature of surroundings
B. Early putrefactive changes
C. Clothings
D. Dehydration

69
258. Which of the following hastens cooling of the body after death?
A. Middle age
B. Asphyxia
C. Acute pyrexial disease
D. Leanness of the body

259. What is the first stage of muscular changes after death?


A. Lividity
B. Muscular irritability
C. Cadaveric rigidity
D. Secondary flaccidity

260. Which of the following organs putrefy early?


A. Heart
B. Liver
C. Kidney
D. Uterus

261. Death by electrocution is to which of the following?


A. Heart failure
B. Shock
C. Asphyxia
D. Renal failure

262. According to the administrative code, an unembalmed dead


body should be buried within how long after death?
A. 12 hours
B. 48 hours
C. 3 days
D. One week

263. When a blow to the forehead cause contusion of the eyeball due
to fracture of the bone at the roof of the orbit, what is the type of
injury incurred?

70
A. Coup
B. Contre-coup
C. Coup contre-coup
D. Locus minoris resistencia

264. Living in very high altitude may cause what type of asphyxial
death?
A. Anoxic
B. Anemic anoxic
C. Stagnant anoxic
D. Histotoxic anoxic

265. What is the most tragic type of child abuser?


A. One time
B. Constant
C. Ignorant
D. Intermittent

266. What is the average time required for death in drowning?


A. 2-5 minutes
B. 10-15 minutes
C. 20-30 minutes
D. 1-2 hours
267. Which of the following acts of physicians is/are punishable as far
as prohibited drugs are concerned?
A. Unlawful prescription
B. Unnecessary prescription
C. Failure to record prescription
D. All of the above

268. Which of the following should be tested in suspected cases of


lead poisoning?
A. Liver
B. Blood
C. Stomach

71
D. Urine

270. Medical Jurisprudence includes:


a. Rights and obligations of charlatans
b. Resolutions, orders and decrees
c. Physician-patient-hospital-paramedical relationship
d. None of the above

271. The right to regulate the practice of medicine by the State is


based on:
a. Police power
b. Power of eminent domain
c. Medical Act of 1959
d. Constitution

272. Qualifications of those appointed members of the Board of


Medicine:
a. He is a duly-registered physician
b. He is a citizen of the Philippines
c. He has been in the practice of Medicine for at least 15
years
d. He is a specialist and not connected with any medical
school

273. Which of the following statements is/are true of faith healing


in relation to the practice of medicine:
a. If a person acted pursuance of his religious belief
and the act is in accordance with the tenets of his
church, it is deemed to be part of his religious
freedom
b. The Medical Act of 1959 exempts faith healing from the
definition of the acts which constitute practice of
medicine

72
c. The faith healer can practice even without a certification
of registration from the Board of Medicine.
d. He can invoke his right to livelihood as the basis for his
performing faith healing

274. Certificate of registration shall not be required on the


following persons:
a. Commissioned medical officers of other countries
stationed in the Philippines within the limit of their
own respective territorial jurisdiction
b. Physicians from other countries called in consultation in
all cases in any hospital after securing a previous
authorization from the Board of Medicine
c. Filipino physicians practicing in foreign countries
d. Dentist in relation to his practice of tooth extraction

275. The following are qualified to practice medicine in the


Philippines:
a. Those who have complied with the pre-requisites to
the practice of medicine in accordance with Sec. 8 of
the Medical Act of 1959
b. Commissioned medical officers of other countries
stationed in the Philippines irrespective of its territorial
jurisdiction
c. Post-graduate medical interns during from-duty hours
d. Those foreign physicians not covered by reciprocity

276. In his rights, a physician may be limited by the dictates of his


conscience, and thus he may refuse which of the following:
a. Bilateral tubal ligations
b. Blood transfusion in emergency cases
c. Prescription of sedatives in psychotic patient
d. Intubation of a severely dyspneic and tachypneic patient

73
277. In which of the following is there a physician-patient
relationship:
a. P.E conducted for insurance purposes with
prescription
b. Physician appointed by trial courts to examine and
accused
c. Autopsy examinations
d. In casual consultations during conferences

278. A physician should never examine or treat hospitalized


patient of another physician without the letters knowledge and
consent except when:
a. In case of emergency, the attending physician is not
readily available
b. The patient expressly asks him to do so in the absence
of attending physician
c. He is a relative of the patient
d. He will not charged the patient a professional fee

279. A patient has the right to discharge the physician treating


him at any time. On the other hand the physician can withdraw
from the physician-patient relationship only if:
a. The patient consents to his withdrawal and is given
ample time to secure the services of another
physician
b. The patient intends to file a suit against him in court
c. There are other physicians in the locality who can treat
the patient
d. The physician lacks the facilities for treatment of the
patient

280. In which of the following is consent necessary:


a. Consultation in an out-patient basis
b. Compulsory procedure by law, e.g. vaccination

74
c. In extreme emergencies
d. Waiver on the part of the patient

281. Dr. Francis Duke, a government physician may not practice


his profession within office hours. This is a limitation imposed
by:
a. Law
b. Religion
c. Hospital Rules and Regulations
d. PMA

282. Which one of the following refers to the inherent right of a


physician:
a. To choose patients
b. To perform certain services
c. To charge professional fees
d. To hold certain public office

283. The Doctrine of Superior Knowledge states that:


a. The physician has superior knowledge over his
patient
b. The patient follows the instructions of the physician
c. The patient determines the proper course of action in the
management
d. The physician is in command and control over the patient

284. The receptionist in the hospital lobby receives a fee from Dr.
Boll for referring cases to the latter. This fee, which is unethical,
is called:
a. Dichotomous fee
b. Contingent fee
c. Retainer fee
d. Referral fee

75
285. Which one of the following is measured by the space of time
provided by the physician and not by the quality and quantity
of medical services rendered:
a. Retainer fee
b. Package deal fee
c. Contractual fee
d. Simple fee
286. By refusing to allow medical students to see her in her
room, a patient is practicing her right to;
a. Privacy
b. Confidentiality
c. Refuse treatment
d. Disclosure
287. It is a common practice of hospitals to insert this clause for
the purpose of making the patient assume the risk of operation:
a. Exculpatory clause
b. Conditional consent
c. Blanket consent
d. General clause

288. You evaluated a 65 year old man with 3-month history of


chest pains and fainting spells that you feel merit cardiac
catheterization. After informing him fully of the benefits,
consequences, risks, etc. he was able to demonstrate that he
understands all of these and refuses the intervention. What
should you do?
a. Respect his choice
b. Explore reasons for his decision and try to convince him
again
c. Consider the discussion ended
d. Continue with the procedure nevertheless

289. A 60-year old patient has complaints of abdominal pain that


is persistent but not extreme. Work-up shows hepatoma. He

76
just retired from a busy professional career and he and his wife
are about to leave for a round the world cruise. What should
you do?
a. Inform him of his condition now
b. Exercise the therapeutic privilege and inform him later
when he returns
c. Inform him when his condition worsens
d. Inform his wife first

290. Audric, an 18 year-old married male still living and


dependent with his parents, was diagnosed with ruptured acute
appendicitis. Who should give the consent for surgery?
a. Audric
b. Parents
c. Pauls wife
d. Grandparents
291. Confidentiality of information in the course of a doctor-patient
relationship may be breached in the following instances:
a. When disclosure is necessary to serve the best
interest of justice
b. When disclosure of information will serve both public and
private health and safety
c. When disclosure is needed in a civil case to determine
the moral character of a patient
d. When information are being asked by police authorities

292. A resident physician followed the order of a consultant that


was grossly wrong and resulted in injury to the patient.
Which is correct?
a. Both the consultant and the resident are liable for
poor patient care
b. The resident clerk is not liable since he is merely a
trainee under the supervision of the consultant.

77
c. The consultant is not liable since the discretion lies with
resident to either follow or not to follow his advice
d. The resident is liable since he is the one who exercised
the clinical judgment that injured the patient

293. Jerichos inability to pay Dr. Abad his fees for services
rendered may result in:
a. Action for collection filed by the physician against
the patient
b. Automatic termination of the patient-MD relationship
c. Lessened physician liability for negligence
d. Abandonment
294. The following are acts considered as immoral or
dishonorable conduct under the Code of Medical Ethics:
a. Fee splitting
b. Insanity
c. Drug Addiction
d. Conviction of a crime

295. These are damages imposed on a physician as


punishment and to serve as an example or correction for the
medical profession:
a. Exemplary damages
b. Nominal damages
c. Compensatory damages
d. Temperate damages
296. The venue where to file the complaint for immorality against
physician is:
a. Professional Regulation Commission
b. Regional Trial Court
c. Prosecutors Office
d. PMA
297. Dr. Luthor performed a surgery on Mr. Kent despite not
having the proper training. Negligently, he nicked the colon and

78
cut the ureter while removing the appendix. Mr. Kent asked Dr.
Luthor for monetary compensation. The nature of liability
against Dr. Jose would be:
a. Civil
b. Administrative
c. Criminal
d. Ethical
298. Dr. Luthor performed a surgery on Mr. Kent despite not
having the proper training. Negligently, he nicked the colon and
cut the ureter while removing the appendix. Mr. Kent asked Dr.
Luthor for monetary compensation. The penalty for the
negligence of Dr. Luthor would be in the nature of:
a. Damages
b. Imprisonment
c. Suspension to revocation of license
d. Expulsion from membership with the PMA

LEGAL/JURIS MEDICINE

1. Patient was aware that physician was drunk yet he allowed the
doctor to inject him:
A. doctrine of continuing negligence C. res ipsa loquitor
B. vicarious liability D. captain of the ship
2. Patient Mr. Lee promised to pay Dr. B one million pesos if he us
cured of his cancer. The professional fee to be collected by B refers
to:
A. contingent fee B. package deal fee C. retainers fee
D. simple contractual fee
3. HIV patient takes experimental drugs despite unknown side
effects:
A. doctrine of informed consent C. assumption of risk
B. doctrine of foreseeability D. doctrine of
contributory negligence
4. There is direct physical connection between the wrongful act of the
Physician and the injury suffered by the patient:

79
A. Doctrine of proximate cause C. Sine Qua now test
B. But for test D. Substantial factor test
5. Principle applied when the medical fee is not specified:
A. Asumpsit on quanthum merit C. Dura Lex sed Lex
B. Lex Loci contract D. Sine Qua now
6. Article 365 which deals with imprudence and negligence is a
provision of:
A. Civil code B. Medical Act 1959 C. Revised
penal code D. Criminal Act of 1970
7. To which forum can an administrative complaint be filed against
doctors?
A. Regional Trial Court C. Department of health
B. National Bureau of Investigation D. Professional
Regulation Comission
8. Reprimand is a sanction imposed against a doctor when he is
guilty of what liability:
A. Administrative liability B. Civil liability C. criminal liability
D. no liability
9. If the license of a physician is suspended indefinitely, after how
long can he apply for reinstatement?
A. 3 years B. 6 months C. 2 years D. it will depend on the pleasure
of the Board of medicine
10. What is the degree of proof/evidence needed to convict physician
of a crime?
A. Guilt beyond reasonable doubt C. Prosecution must prove
that Dra. Gajardo got pregnant
B. Preponderance of evidence D. Prosecution must prove that Dr.
Gimenez is a medical doctor
11. The study of licensure laws, regulatory laws and physician-patient
relationship, is known in medical school curriculum as:
A. Legal Medicine B. Legal Ethics C. Medical
Jurisprudence D. M<edical Ethics
12. Which of the following does not affect a termination of physician-
patient relationship?

80
A. withdrawal of physician without patient consent C.
death of patient
B. recovery of patient D. death of
physician
13. A physician should never examine or treat hospitalized patient of
another physician without the latters knowledge and consent except
when:
A. the patient expressly asks him to do so in the absence of
attending physician
B. he is a relative of the patient
C. he will not changed the patient a professional fee
B. in case of emergency, the attending physician is not
readily available
14. A physician accused of criminal abortion was acquitted because:
A. he was a duly licensed medical practitioner
B. the woman plaintiff was emotionally unstable
C. the product of conception could not be produced in
court
C. a pregnancy test done 3 days before the alleged abortion
definitely ruled out pregnancy

CASE: Mr. A brought his 3 yrs old son to a hospital. Dr. B, the
admitting physician admitted the pediatric case to the medicine ward
because the pediatric ward is full. During the night, the patient child
was seen in the floor with a fractured skull resulting from a fall. It was
found out that the bed was without railings. Mr. A wanted to sue Dr. B
and the hospital.
15. The doctrine applicable in this case would be:
A. Res ipsa loquitor B. Osiensible agent C. Forseeability
D. Common knowledge
16. Suppose before admission Mr. A was informed of the absence of
railings to protect the child from fall: A. assumption of risk
C. res ipsa loquitor
B. continuing negligence D. common knowledge

81
17. Suppose Mr. A has already previously and repeatedly complained
of the absence of railings to Dr. B but no attention was given to the
complaint, which doctrine could be applicable?
A. assumption of risk C. contributory negligence
B. continuing negligence D. common knowledge
18. Suppose one day after an intramuscular injection a part of the
needle was discovered protruding from the buttocks of the patient,
what is the doctrine applicable?
A. assumption of risk B. res ipsa loquitor C. contributory
negligence D. continuing negligence

19. Originally there were railings on the bed of the patient. Mr A, the
father removed the railings so that he himself can sleep on the bed.
His child fell with a broken skull. What is the doctrine applicable: A.
res ipsa loquitor C. contributory negligence
B. borrowed servant doctrine D. common knowledge
20. Patient leaves the hospital against advice:
A. Continuing Negligence C. Vicarious Liability
B. Res Iplsa Loquitor D. Contributory
Negligence
21. Common defense of the hospital are regards consultants liability
to his private patients:
A. Independent Contractor C. Captain of the ship
doctrine
B. Joint and several liability D. Vicarious liability
22. The reporting of treatment of serious physical injuries by a
physician is:
A. Voluntary B. Mandatory C. Case to case D. Every
end of the month
23. Under the doctrine the hospital owner may be liable for the
mistakes of the residents:
A. Borrowed servant doctrine C. Vicarious liability

82
B. Captain of the ship D. Fellow Servant doctrine
24. Which of the following laws is enforced by the state:
A. Substantive law B. Moral law C. Divine Law
D. Natural Law
25. A terminally ill patient is given an overdose of sedatives without
his knowledge. What crime if any is committed by the doctor:
A. Murder C. Homicide
B. No crime because ethically allowed D. Assisted Suicide
26. The bone which is most informative regarding the sex of the
owner of the skeleton being examined is the: A. Skull B.
Sternum C. Pelvis D. Femur
27. The willful deliberate and painless acceleration of death of a
person:
A. Parricide B. Euthanasia C. Homicide D.
Infanticide
28. The cause of death resulting from lack of skill, lack of foresight,
and reckless imprudence is called: A. Homicide B. Accidental
C. Murder D. Negligence
29. When a body is the subject matter of a criminal investigation, it
may be exhumed:
A. Anytime B. 3 years after burial C. 5 years after
burial D. 10 years after burial
30. Any change for the position of the cadaver will not change the
location of this type of cadaveric lividity: A. Hypostatic B. Osmotic
C. Diffusion D. Alteration
31. Exhumation is also known as:
A. interment B. ground burial C. embalming
D. disinterment
32. Deep burial sites will make decomposition :
A. faster B. slower C. stationary
D. negligible
33. When blood accumulates in the most dependent portions of the
dead body, it is called
A. primary6 flaccidity C. post-mortem
lividity

83
B. secondary flaccidity D. algor mortis
34. A missing person is presumed dead if he is not heard from for:
A. 1 year B. 2 years C. 3 years D. 4 years
35. Musketry, electrocution, and hanging are examples of
A. dead body disposal C. sudden death
B. judicial death D. asphyxial death
36. If the physician performs euthanasia without the patients
knowledge, he is guilty of:
A. murder C. merciful killing
B. nothing D. assistance to the
commission of suicide
37. If a 3-day old unidentified child was intentionally killed with
cruelty, and during a calamity, the offender may be changed with:
A. infanticide B. murder C. homicide
D. parricide
38. Cadaveric rigidity is also known as
A. Rigor mortis C. cadaveric spasm
B. Algor mortis D. heat stiffening
39. The convict is put to death by a volley of fire from a firing squad
in:
A. mugging B. Muskety C. garroting
D. murder
40. An unembalmed body should be buried not later than: (if the
person died as a result of a natural cause): A. 12 hrs B. 20 hrs.
C. 24 hrs. D. 48 hrs.
41. An unembalmed body should be buried not later than: (if the
person died as a result of a communicable disease): A. 12 hrs.
B. 20 hrs. C. 24 hrs. D. 48 hrs.
42. If one performed lawful act, and he did it with due care, and death
occurred without intending to cause it, the death is
A. negligent B. homicide C. accidental
D. murder
43. Instantenous rigor is also known as
A. Rigor mortis C. post-mortem muscular
irritability

84
B. cadaveric spasm D. post-mortem rigidity
44. It is the prominence of the superficial veins with reddish
discoloration due to the process of decomposition which develops on
both flanks of the abdomen.
A. putrefaction C. Adiposer formation
B. Marbolization D. Maceration

45. The microorganisms that plays a dominant role in decomposition


is
A. bacillus mesenterious C. chlostridium
welchi
B. bacillus coli D. bacillus proteus
vulgaris
46. The following manner of death should be autopsied, EXCEPT
A. suicides C. accidental death
B. death unattended by physician D. death occurring in
natural manner
47. A common law-husband who kills his common law wife with
whom he has a 2 days old child can be prosecuted for
A. infanticide C. homicide
B. parricide D. impossible crime
48. Presidential Decree 169 mandates all doctors to report all cases
involving:
A. slight and less physical injury C. less serious and
serious physical
B. mutilation D. child abuse cases
49. When the color of the contusion changes to green, the
contusions estimated age is
A. 2-4 days B. 4-5 days C. 7-10 days
D. 14-15 days
50. Fracture produced by a tangential or glancing approach of bullet:
A. pond fracture C. gutter fracture
B. indented fracture D. bursting fracture

85
51. This is a firearm which has a cylindrical magazine situated at the
rear of the barrel which can accommodate five or six cartridge:
A. automatic pistol C. rifle
B. revolver D. shotgun
52. Its main function is the transformation of mechanical energy by
the hit of the firing pin on the percussion cap to chemical energy:
A. primer B. powder C. bullet D.
projectile
53. If the diameter of the gunshot wound point of entry is 1 cm x 1 cm
the probable caliber of the firearm used is
A. 0.38 B. 0.22 C. 0.45 D. 9 mm
54. The primer is usually made up of
A. lead, barium, antimony C. lead and antimony
B. lead and barium D. lead, barium and
copper
55. Tattooing is also known as
A. stippling B. soot C. smudging
D. fouling
56. Factors which make the wound of entrance bigger than the
caliber, except
A. deformity of the bullet which entered
B. bullet enters the skin sidewise
C. acute angular approach of the bullet
D. contraction of the elastic tissues of the skin
57. Pugilistic position of a boxer is associated with
A. burns B. drowning C. asphyxia
D. gunshot wounds
58. The average time required for death in drowning is
A. 1-3 minutes C. 5-10 minutes
B. 2-5 minutes D. 3-6 minutes
59. This is quantitative determination of the chloride content of the
blood in the right and left ventricle of the heart:
A. Gettlers test C. Winslows test
B. Takayama test D. Loords test

86
60. The test which determines the whether semen is of human origin
or not:
A. biological test of farnum C. Berberios
B. Gangulis method D. puramen reaction
61. It is an expressed acknowledgement by the accused in a criminal
case of the truth of his guilt as the crime charged.
A. confession D. all of the above
B. admission E. none of the above
C. interrogation
62. The presence of weapon which is highly grasped by the hand of a
victim of a shooting incident is an example of
A. Rigor mortis C. cadaveric spasm
B. death stiffening D. cadaveric rigidity
63. Cases when trauma or disease kill quickly that there is no
opportunity for sequel or complication to develop is known as
A. proximate cause of death C. immediate cause
of death
B. secondary cause of death D. complete cause of
death
64. The most practical, useful and reliable means of preserving
evidence is by
A. photography C. description
B. sketching D. manikin method
65. The comprehensive study of a dead body performed by trained
physician employing recognized procedures and techniques is known
as
A. Post-mortem examination C. medical examination
B. autopsy D. Post-mortem dissection

66. The principle used in the Magnus test for determining somatic
death is
A. no change in color of the finger
B. no change in the temperature of the finger
C. change in the color of the finger
D. change in the temperature of the torniqueted finger

87
67. When the body surface of corpse is pressed it leaves a flattened
area due to
A. absence of edema C. precipitated skin
proteins
B. loss of elasticity of the skin D. absence of blood
68. Cutis Galina or Washer womans hands and feet on the corpse
signifies that:
A. the body has been in water for sometime
B. the body is that of a laundry woman
C. the body died for drowning
D. the body died for syphilis
69. Permission to disinter remains of persons who died of dangerous
communicable diseases maybe granted after a burial period of
A. 4 years B. 5 years C. 6 years D. 7 years
70. This is a medical evidence made known or addressed to the
senses of the court no limited to the sense of vision, but is intended
to the sense of hearing, taste, smell and touch
A. experimental evidence C. autoptic or real
evidence
B. documentary evidence D. corpus evidence
71. The most prominent sign of death is
A progressive fall of the body temperature
B. cessation of heart action and circulation
C. cessation of respiration
D. insensibility of body loss of power to move
72. The study of licensure laws, regulatory laws and physician-patient
relationship, is known in medical school curriculum as:
A. Legal Medicine C. Medical Jurisprudence
B. Legal Ethics D. Medical Ethics
73. A mental process whereby one or more persons appraise a
situation and make a decision based on their judgement that may or
may not lead to action is called:
A. Treatment B. Diagnosis C. Operation D.
Analysis

88
74. Faith healing is allowed as an norm of treatment modality in the
Philippines. This is allowed on the basis of Constitutional guarantee
on:
A. freedom of origin C. freedom of the press
B. freedom of expression D. freedom of sexual
expression
75. The nature of liability against Dr. a would be:
A. administrative B. civil C. criminal D. ethical
76. The venue with which to file the case in case Mr. Ty demands P1
million pesos as payment would be: A. Regional Trial Court
C. Department of health
B. Professional Regulation commission D. All of the
above
77. The cost of the procedure to re-attach of Mr. Ty that was incided
would be in nature of:
A. moral damages C. exemplary
damages
B. actual damages D. all of the above
78. Mr. Ty suffered sleepless nights and wounded feelings as well as
besmirched reputation.
This kind of damage is referred to as:
A. actual damage C. exemplary damage
B. moral damage D. attorneys fees
79. Dr. A uttered vexing words against Dr. B defaming his personality
and dignity. Dr. A maybe guilty of:
A. Libel B. Slander C. rumor mongering D. any of the
above
80. When a surgeon operates on a patient without consent, this
constitutes an assault and even without proof of injury resulting, the
plaintiff may recover what type of damage?
A. Nominal B. Moderate C. Exemplary D. Liquidated
81. When is patients consent necessary?
A. in an emergency, patient need tracheostomy

89
B. in a women diagnosed with ectopic pregnancy, it turns out
be a case of acute appendicitis and the surgeon decides to
remove the appendix
C. During an appendectomy, the surgeon discovers and
abnormal ovary and desiced to remove it.
D. The colon was perforated during simoidoscopy and the
surgeon decides on laparotomy to undertake repair
82. A physician may perform a diagnostic or therapeutic procedure
without the consent of the patient or his relatives.
A. when the physician believes the procedure is necessary
B. when the patient is merely being stubborn
C. when the patient is unconscious
D. in an acute emergency when there in no time to discuss
the situation in order to seek consent

83. A physician may apply a procedure even it is against the wishes


of the patient if
A. the procedure is known to be safe
B. the procedure is necessary for the good of the patient
D. the procedure is required by law
C. there is implied consent by he patients spouse
84. The following are provisions of Penal Law wherein a physician
may be hold criminally liable. Which provision specifically mention the
physician as the wrong-doer?
A. defamation
B. criminal negligence and imprudence
C. violation of the Dangerous Drug Act
E. refusal to render treatment in emergency cases
85. The testimony of an expert witness is NOT needed to prove a
negligent act when which doctrine is applicable? A. res ipsa
loquitor C. ostensible agent
B. borrowed servant D. captain of the ship
86. Instances when a physician may NOT be held liable for
abandonment.

90
A. failure to visit the patient sufficiently after believing that
the patient needed no further treatment
B. failure to provide follow-up attention
C. refusal to attend a case for which he has assumed
responsibility
D. failure to arrange for a substitute during the attending
physicians absence
87. A physician cannot be held liable for the ill effect of his
management procedure if
F. he can show that he applied his knowledge and skill
with diligence and care
A. the doctrine of res ipsa loquitor is applicable
B. the patient contributed to the ill-effects
C. the procedure is by its nature risky
88. A physician who gives an overdose of a drug may be charged for
A. Immorality C. gross negligence
B. Dishonorable conduct D. incompetence
89. One of the grounds for the suspension or revocation of the
certificate of registration of a physician on account of a criminal act is
A. immoral or dishonorable conduct
G. violation of the Code of ethics
B. conviction by court of an offense involving moral turpitude
C. false, extravagant or unethical advertisement
90. A balikbayan physician puts up a sign board at his clinic. Which is
unethical?
A. Disease of Women and Children C. consultation hours
9-11 a.m.
B. Trained at Mayo Clinic D. consultation by
appointment only
91. The mistress of a married physician sought redress from the
Board of Medicine when the physician left her for another woman.
Which statement is VALID?
A. she can charge the physician for immorality C. she has
no right to complain

91
B. she can accuse him for adultery D. she can
charge him of abandonment
92. An unmarried physician was discovered having sex with an
unmarried nurse. He may be charged before the Board of medicine
for
A. Seduction B. Immorality C. Adultery D. reckless
imprudence
93. The prohibition and imposition of penalty on physicians who sell
medical samples gratuitously given to them by drug promoters is
embodied in
A. Medical Act 1959 as amended C. Revised Penal
Code
B. Revised Administrative Code D. Pharmacy Law
94. The element of deceit is required in case of:
A. forcible obduction C. simple seduction
B. consented obduction D. qualified seduction
95. The spinning motion of the bullet, its rough surface, and the
imagination of the skin, all produce the: A. Tattooing B. Rifling
C. Contusion collar D. smudging
96. The skin is whitened, macerated and wrinkled in:
A. cutis anserina C. washerwomans skin
B. saponification D. contact flattening
97. Sexual intercourse is NOT necessary in this crime:
A. Abduction B. Seduction C. Adultery D.
Concubinage
98. Virginity is a requirement in this crime:
A. simple seduction C. qualified seduction
B. forcible abduction D. white slave trade
99. The laceration involving more than half of the hymenal height but
does not reach the hymenal based is called:
A. Superficial B. Deep C. Complete
D. Complicated
100. Stippling is caused by:
A. Flame B. gunpowder residue C. smoke D.
bullet

92
REFFERENCE BOOK: SOLIS

93

You might also like